Tải bản đầy đủ (.pdf) (64 trang)

Tuyển chọn một số bài toán bất đẳng thức thức hay

Bạn đang xem bản rút gọn của tài liệu. Xem và tải ngay bản đầy đủ của tài liệu tại đây (1.2 MB, 64 trang )

Tailieumontoan.com
TUYỂN CHỌN VÀ GIỚI THIỆU MỘT SỐ BẤT ĐẲNG THỨC HAY
Bài 1. Cho a, b, c là các số thực dương thỏa mãn

T=

nhất của biểu thức:

1 1 1
+ +  a + b + c . Tìm giá trị lớn
a b c

1
1
1
+
+
2
2
2 + a 2 + b 2 + c2
Lời giải

Dự đoán giá trị lớn nhất T là 1 và xẩy ra khi a = b = c = 1 . Như vậy ta cần chứng minh
được bất đẳng thức T =

1
1
1
+
+
 1 . Thật vậy, bất đẳng thức cần chứng


2
2
2 + a 2 + b 2 + c2

minh tương đương với

1
1
1
1
1
1
a2
b2
c2

+

+


1

+
+
1
2 2 + a 2 2 2 + b2 2 2 + c 2
a 2 + 2 b2 + 2 c 2 + 2
Áp dụng bất đẳng thức Cauchy – Schwarz dạng phân thức ta có


(a + b + c )
a2
b2
c2
+
+

a 2 + 2 b2 + 2 c 2 + 2 a 2 + b2 + c 2 + 6
2

Phép chứng minh sẽ hoàn tất nếu ta chỉ ra được

(a + b + c )

2

a +b +c +6
2

2

2

 1  ( a + b + c )  a 2 + b2 + c 2 + 6  ab + bc + ca  3
2

Để ý ta viết lại giả thiết thành ab + bc + ca  abc ( ab + bc + ca ) .
Mà ta có abc ( ab + bc + ca )

Do đó ta có


(ab + bc + ca )
3

( ab + bc + ca )

3

2

.

2

 ab + bc + ca  ab + bc + ca  3 . Như vậy bất đẳng thức

được chứng minh. Vậy giá trị lớn nhất của T là 1, xẩy ra tại a = b = c = 1 .
Bài 2. Cho a, b, c là độ dài ba cạnh của một tam giác thỏa mãn a + b + c = 1 . Chứng
minh rằng:

b+c −a c +a − b a + b−c
+
+
4
a 2 + bc b2 + ca c 2 + ab
Lời giải
Trước hết ta pháp biểu bất đẳng thức phụ. Cho hai bộ số a, b, c và x, y, z thỏa mãn
điều kiện a  b  c; x  y  z . Khi đó ta ln có ax + by + cz 
Nguyễn Công Lợi


1
(a + b + c )( x + y + z ) .
3

Trường THCS Thị Trấn Quỳ Hợp – Nghệ An


Tailieumontoan.com
Trở lại bài toán. Do a + b + c = 1 nên bất đẳng thức cần chứng minh tương đương với

( b + c − a )( a + b + c ) + ( c + a − b )(a + b + c ) + (a + b − c )(a + b + c )  4
a 2 + bc

( b + c)


2

( b + c)


2

( b + c)


2

− a2


a 2 + bc

(c + a)
+

b2 + ca

2

− b2

b2 + ca

− a2

a 2 + bc

(c + a)
+1+

(a + b )
+

c 2 + ab

2

− c2

c 2 + ab


2

− b2

b2 + ca

4

(a + b)
+ 1+

2

− c2

c 2 + ab

+1 7

( c + a ) + ca + ( a + b ) + ab  7
bc
+
+
a 2 + bc a 2 + bc b2 + ca b2 + ca c 2 + ab c 2 + ab
2

2

Mà theo bất cẳng thức Cauchy – Schwarz dạng phân thức ta có


( ab + bc + ca )
bc
ca
ab
+
+

=1
a 2 + bc b2 + ac c 2 + ab bc a 2 + bc + ca b2 + ca + ab c 2 + ab

(

Như vậy ta cần chứng minh được

)

(

)

( b + c ) + ( c + a ) + (a + b)
2

2

2

a 2 + bc


b2 + ca

c 2 + ab

(

)

 6 . Thật vậy bất đẳng

thức tương đương với

(b + c)

2

(c + a)
−2+

2

(a + b )
−2+

2

−20
a 2 + bc
b2 + ca
c 2 + ab

b2 + c 2 − 2a 2 c 2 + a 2 − 2b 2 a 2 + b 2 − 2c 2

+
+
0
a 2 + bc
b2 + ca
c 2 + ab
Khơng mất tính tổng qt ta giả sử a  b  c .
 b2 + c 2 − 2a 2  c 2 + a 2 − 2b2  a 2 + b 2 − 2c 2

Khi đó ta có  1
.
1
1

+
 2
 a + bc b2 + ca c 2 + ab

Áp dụng bất đẳng thức phụ trên thì ta thu được
b 2 + c 2 − 2a 2 c 2 + a 2 − 2b 2 a 2 + b 2 − 2c 2
+
+
a 2 + bc
b 2 + ca
c 2 + ab
1
 1
1

1 
 b 2 + c 2 − 2a 2 + c 2 + a 2 − 2b 2 + a 2 + b 2 − 2c 2  2
+ 2
+ 2
=0
3
 a + bc b + ca c + ab 

(

)

Vậy bất đẳng thức được chứng minh.
Bài 3. Cho a, b, c là các số thực dương bất kỳ. Chứng minh rằng:

bc
+
( c + a )(a + b )

ca
+
( c + b )(a + b )

ab
4abc
 1+
( c + a )( c + b )
(a + b )( b + c )( c + a )

Lời giải


Nguyễn Công Lợi

Trường THCS Thị Trấn Quỳ Hợp – Nghệ An


Tailieumontoan.com

( a + b )( a + c )  a +

Áp dụng bất đẳng thức Cauchy – Schwarz ta có

bc . Do đó ta

được
bc
=
( c + a )( b + a )

bc ( a + b )( a + c )

( a + b )( a + c )



(

bc a + bc

)=


( a + b )( a + c )

abc.

a
bc
+
(a + b )(a + c ) (a + b )(a + c )

Chứng minh hoàn tường tương tự ta có

ca
b
ca
 abc.
+
( a + b )( b + c )
( a + b )( b + c ) (a + b )( b + c )
ab
c
ab
 abc.
+
( a + c )( b + c )
( a + c )( b + c ) (a + c )( b + c )
Suy ra

bc
+

( c + a )(a + b )

ca
+
( c + b )(a + b )

ab
 M + N , trong đó
( c + a )( c + b )



a
b
c
M = abc 
+
+

 ( a + b )( a + c ) ( a + b )( b + c ) ( a + c )( b + c ) 
bc
ca
ab
N=
+
+
( a + b )( a + c ) ( a + b )( b + c ) ( a + c )( b + c )

Mặt khác ta có


bc
ca
ab
2abc
+
+
+
( a + b )( a + c ) ( a + b )( b + c ) ( a + c )( b + c ) (a + b )( b + c )( c + a )
=
=

bc ( b + c ) + ca ( c + a ) + ab ( a + b ) + 2abc

( a + b )( b + c )( c + a )

a 2 b + ab 2 + b 2 c + 2bc 2 + c 2a + ca 2 + 2abc ( a + b )( b + c )( c + a )
=
=1
( a + b )( b + c )( c + a )
( a + b )( b + c )( c + a )

Do đó N = 1 −

2abc
. Như vậy phép chứng minh sẽ hoàn tất neeusta chỉ
( a + b )( b + c )( c + a )

ra được




a
b
c
2abc
abc 
+
+
 +1−
(a + b )( b + c )( c + a )
 ( a + b )( a + c ) ( a + b )( b + c ) ( a + c )( b + c ) 
4abc
 1+
( a + b )( b + c )( c + a )


a
b
c
6abc
 abc 
+
+

 ( a + b )( a + c ) ( a + b )( b + c ) ( a + c )( b + c )  ( a + b )( b + c )( c + a )
 a ( b + c ) + b ( c + a ) + c ( a + b )  6 abc
Nguyễn Công Lợi

Trường THCS Thị Trấn Quỳ Hợp – Nghệ An



Tailieumontoan.com
Dễ thấy theo bất đẳng thức AM – GM thì ta ln có

a ( b + c ) + b ( c + a ) + c (a + b )  2 a. bc + 2 b. ca + 2 c. ab = 6 abc
Vậy bất đẳng thức đã cho được chứng minh, đẳng thức xẩy ra khi và chỉ khi a = b = c .
Bài 4. Cho a, b, c là các số thực dương thỏa mãn a2 + b2 + c2 = 3 . Tìm giá trị nhỏ nhất
của biểu thức:
a3

S=

1 + b2

b3

+

1 + c2

+

c3
1 + a2

Lời giải
Áp dụng bất đẳng thức Cauchy – Schwarz dạng phân thức ta có

S=



a3
1 + b2

b3

+

(a

1 + c2
2

c3

+

+ b2 + c 2

)

a4

=

1 + a2

a 1 + b2

+


b4
b 1 + c2

+

c4
c 1 + a2

2

a 1 + b2 + b 1 + c 2 + c 1 + a 2

=

9
a 1 + b2 + b 1 + c 2 + c 1 + a 2

Lại áp dụng bất đẳng thức ta có

(a

a 1 + b2 + b 1 + c 2 + c 1 + a 2 
=

Từ đó ta có

(a

2


)(

)

)

+ b2 + b2 a 2 + b2 + c 2 + 3 = 3. ( 3 + 3 ) = 3 2

9
a 1+ b + b 1+ c + c 1+ a
2

)(

+ b2 + b2 a 2 + 1 + b2 + 1 + c 2 + 1

2

2

2



9
3 2

=


3 2
3 2
suy ra S 
, dấu bằng xẩy
2
2

ra khi và chỉ khi a = b = c = 1 .
Vậy giá trị nhỏ nhất của S là

3 2
, xẩy ra khi a = b = c = 1 .
2

Bài 5. Cho x, y, z là các số thực thỏa mãn x + y + z = 0 . Chứng minh rằng:

x (x + 2)
2x + 1
2

+

y ( y + 2)
2y + 1
2

+

z (z + 2)
2z 2 + 1


0

Lời giải
• Xét trường hợp xyz = 0 , khi đó kết hợp với giả thiết ta suy ra được một trong các

trường hợp sau xẩy ra x = 0; y = −z hoặc y = 0; x = −z hoặc z = 0; x = −y .
Nếu x = 0; y = −z thì bất đẳng thức cần chứng minh trở thành

y ( y + 2)
2y 2 + 1
Nguyễn Công Lợi

+

y ( y − 2)
2y 2 + 1

0

y 2 + 2y + y 2 − 2y
 0  y2  0
2
2y + 1

Trường THCS Thị Trấn Quỳ Hợp – Nghệ An


Tailieumontoan.com
Bất đẳng thức cuối cùng đúng nên bất đẳng thức đã cho được chứng minh.

Dấu bằng xẩy ra khi và chỉ khi x = y = z = 0 .
Với y = 0; x = −z hoặc z = 0; x = −y chứng minh hồn tồn tương tự.
• Xét trường hợp xyz  0 , khi đó do x + y + z = 0 nên tồn tại hai số cùng dấu, khơng

mất tính tổng qt ta giả sử hai số đó là y và z, khi đó ta có yz  0 . Bất đẳng thức cần
chứng minh tương đương với

2x ( x + 2 )

+1+

2x + 1
2



2y ( y + 2 )
2y + 1
2

2z ( z + 2 )

+1+

2z 2 + 1

+1 3

2x 2 + 4x + 2x 2 + 1 2y 2 + 4y + 2y 2 + 1 2z 2 + 4z + 2z 2 + 1
+

+
3
2x 2 + 1
2y 2 + 1
2z 2 + 1

( 2x + 1) + ( 2y + 1) + ( 2z + 1)

2

2

2x 2 + 1

2y 2 + 1

2

3

2z 2 + 1

Để ý rằng x + y + z = 0 và yz  0 . Áp dụng bất đẳng thức Cauchy – Schwarz dạng
phân thức ta có

( 2y + 1) + ( 2z + 1)
2

2y 2 + 1


2

2z 2 + 1



2 ( y + z + 1)
y2 + z2 + 1

2 ( x − 1)

2

=

( y + z)

2

2

− 2yz + 1

=

2 ( x − 1)

x 2 − 2yz + 1

( 2x + 1)


Như vậy phép chứng minh sẽ kết thúc nếu ta chỉ ra được

2

2

2x2 + 1

+



2 ( x − 1)

2

x2 + 1

2 ( x − 1)
x2 + 1

2

 3.

Bất đẳng thức trên tương đương với

( 2x + 1) ( x + 1) + 2 ( x − 1) ( 2x + 1)  3 ( 2x + 1)( x + 1)
 ( x + 1) ( 2x + 1) − 3 ( 2x + 1)  + 2 ( x − 1) ( 2x + 1)  0



2

2

2

2

2

(

)

2

2

2

2

(

2

2


)

 x 2 + 1  −2x 2 + 4x − 2  + 2 ( x − 1) 2x 2 + 1  0

(

2

)

(

)

 − x 2 + 1 ( x − 1) + 2 ( x − 1) 2x 2 + 1  0  ( x − 1) x 2  0
2

2

Bất đẳng thức cuối cùng luôn đúng nên ta có

( 2x + 1)

2

2

2x2 + 1

+


2 ( x − 1)
x2 + 1

2

 3.

Vậy bất đẳng thức đã cho được chứng minh. Dấu bằng xẩy ra khi và chỉ khi

( x; y; z ) =  1; − 21 ; − 21  và các hoán vị.




Bài 6. Cho a, b, c là các số thực thỏa mãn a2 + b2 + c2 = 8 . Chứng minh rằng:

(a − b) + ( b − c ) + ( c − a )
5

Nguyễn Công Lợi

5

5

 960

Trường THCS Thị Trấn Quỳ Hợp – Nghệ An



Tailieumontoan.com
Lời giải
Để ý rằng a − b + b − c + c − a = 0 . Đặt a − b = x; b − c = y suy ra c − a = − ( x + y )
Từ đó ta có phép biến đổi

(

)

x 5 + y 5 − ( x + y ) = − x 5 + 5x 4 y + 10x 3 y 2 +10x 2 y 3 + 5xy 4 + y 5 + x 5 + y 5
5

(

)

(

= − 5x 4 y + 10x 3 y 2 + 10x 2 y 3 + 5xy 4 = −5xy x 3 + 2x 2 y + 2xy 2 + y 3

(

)

(

)

= −5xy ( x + y ) x 2 − xy + y 2 + 2xy ( x + y )  = −5xy ( x + y ) x 2 + xy + y 2




)

Như vậy ta có

(a − b) + ( b − c ) + (c − a )
5

5

5

(

= 5 ( a − b )( b − c )( c − a ) a 2 + b 2 + c 2 − ab − bc − ca

)

Đặt A = ( a − b ) + ( b − c ) + ( c − a ) , khi đó kết hợp với a2 + b2 + c2 = 8 ta có
5

5

5

(

A = 5 ( a − b )( b − c )( c − a ) a 2 + b2 + c 2 − ab − bc − ca

= 5 ( a − b )( b − c )( c − a )( 8 − ab − bc − ca )

)

Suy ra A2 = 25 ( a − b ) ( b − c ) ( c − a ) ( 8 − ab − bc − ca ) .
2

2

2

2

Do vai trị của a, b, c như nhau nên ta có thể giả sử a  b  c .
Khi đó ta có ( a − b ) ( b − c ) = ( a − b )( b − c )
2

2

2

(a − c )


4

16

A2 = 25 ( a − b ) ( b − c ) ( c − a ) ( 8 − ab − bc − ca ) 
2


2

2

2

. Như vậy ta có
6
2
25
a − c ) ( 8 − ab − bc − ca )
(
16

Đặt t = ab + bc + ca . Ta có t = ab + bc + ca  a2 + b2 + c2 = 8 . Lại có

(a + b + c )

2

= a 2 + b2 + c 2 + 2 ( ab + bc + ca ) = 2t + 8  0  t  −4

Như vậy ta có −4  t  8 . Mặt khác ta có ( a − b ) + ( b − c ) 
2

Do đó ( a − b ) + ( b − c ) + ( c − a ) 
2

2


2

2

2
2
1
1
a − b + b − c ) = (a − c )
(
2
2

2
3
a − c ) nên ta được
(
2

(

)

2 a 2 + b2 + c 2 − ab − bc − ca 

2
2
3
32 − 4t

a − c )  (a − c ) 
(
2
3

3

2
5
25  32 − 4t 
100
8 − t) =
8 − t) .
Như vậy ta suy ra được A  
(
(

16  3 
27
2

Do −4  t  8 nên suy ra 0  8 − t  8 − ( −4 ) = 12 . Từ đó 0  A2 

100 5
.12  A  960 .
27

Vậy ta có ( a − b ) + ( b − c ) + ( c − a )  960 . Bất đẳng thức được chứng minh.
5


Nguyễn Công Lợi

5

5

Trường THCS Thị Trấn Quỳ Hợp – Nghệ An


Tailieumontoan.com
Bài 7. Cho a, b, c là các số thực dương. Chứng minh rằng:

(

) + 9 (a + b + c )

2 a 3 + b3 + c 3

2

a 2 + b2 + c 2

abc

 33

Lời giải
Bất đẳng thức đã cho có dạng đồng bậc nên khơng mất tính tổng quát ta có thể
chọn a + b + c = 1 . Ta có các phép biến đổi sau


(

a 3 + b 3 + c 3 = ( a + b + c ) − 3 a 2 b + ab 2 + b 2 c + bc 2 + c 2a + ca 2 + 2abc
3

)

= ( a + b + c ) − 3 ( a + b + c )( ab + bc + ca ) − abc 
3

a 2 + b 2 + c 2 = ( a + b + c ) − 2 ( ab + bc + ca )
2

Khi đó bất đẳng thức đã cho được viết lại thành
3
2
2 ( a + b + c ) − 3 ( a + b + c )( ab + bc + ca ) − 3abc 
9 (a + b + c )

+
 33
2
abc
a
+
b
+
c

2

ab
+
bc
+
ca
(
) (
)

Đặt x = a + b + c; y = ab + bc + ca; z = abc , theo cách chuẩn hóa trên ta có x = 1 . Khi đó
bất dẳng thức trên được viết lại thành

2 (1 − 3y + 3z )
z

+

2 (1 − 3y )
9
9
 33 
+
 27 .
1 − 2y
z
1 − 2y

Dễ thấy theo bất đẳng thức AM – GM ta có ( a + b + c )( ab + bc + ca )  9abc .
Do đó ta được xy  9z  z 


2 (1 − 3y )
18 (1 − 3y )
xy y
9
9
.
+

+
= nên ta có
z
1 − 2y
y
1 − 2y
9 9

Phép chứng minh sẽ hoàn tất nếu ta chỉ ra được

(a + b + c )
Thật vậy chú ý rằng ab + bc + ca 
3

18 ( 1 − 3y )
y

+

18 (1 − 3y )

2


y

y

+

9
 27 .
1 − 2y

1
, từ đó ta có
3

9
 27  2 (1 − 3y )(1 − 2y ) + y  3y (1 − 2y )
1 − 2y

 18y 2 − 12y + 2  0  2 ( 3y − 1)  0
2

Bất đửng thức cuối cùng luôn đúng nên bất đẳng thức đã cho được chứng minh. Dấu
bằng xẩy ra khi và chỉ khi a = b = c .
Bài 8. Cho a, b, c là các số thực dương thỏa mãn điều kiện ab + bc + ca  abc . Tìm giá
trị nhỏ nhất của biểu thức:
Nguyễn Công Lợi

Trường THCS Thị Trấn Quỳ Hợp – Nghệ An



Tailieumontoan.com
ab
bc
ca
P=
+
+
c ( ac + bc )
a ( ab + ac )
b ( ab + bc )
Lời giải
Từ ab + bc + ca  abc suy ra

1 1 1
+ +  1 . Đặt
a b c

a=

1
1
1
; b = ; c = , khi đó giả thiết
x
y
z

của bài tốn được viết lại thành x2 + y2 + z2  1 . Ta có biểu thức P trở thành


1
x y2

1
y z2

1
2
y3
x3
z3
z
x
P=
+
+
= 2
+
+
2
2
2
2
2
1 1
1  1 1
1  1 1
1  y +z z +x x +y
+
+

+






z  x2 z2 y2 z2  x  x2 y2 x2 z2  y  x2 y 2 y 2 z2 
2

2

2

Áp dụng bất đẳng thức Cauchy – Schwarz dạng phân thức ta có
y3
y4
x3
z3
x4
z4
+
+
=
+
+
y2 + z2 z2 + x2 x2 + y2 x y2 + z2
y z2 + x2
z x2 + y2


(



)

(x

(

2

(

) (

+ y2 + z2

) (

)

)

2

) (

x y2 + z2 + y z2 + x2 + z x2 + y2


)

Mặt khác cũng áp dụng bất đẳng thức Cauchy – Schwarz ta lại có

(

) (
) (
)
= x ( y + z ) . y + z + y ( z + x ). z + x + z ( x + y ). x + y
  x ( y + z ) + y ( z + x ) + z ( x + y )  . ( y + z ) + ( z + x ) + ( x + y ) 

 

(x + y + z ) . x + y + z
= 2 ( x y + y z + z x )( x + y + z )  2
(
)
3
x y2 + z2 + y z2 + x2 + z x2 + y 2
2

2

2

2

2


2

2

2

2

2

2

2

2

2

2

2

2

2

2

2


2

2

2

2

2

2

=2

2

2

(

1 2
x + y2 + z2
3

(x

Như vậy ta có P 

2


2

)

2

2

2

2

2

2

2

2

2

2

2

2

2


2

2

2

2

2

2

+ y2 + z2

(

)

2

1 2
x + y2 + z2
3

)

=
2

3 2

3
x + y2 + z2 =
. Dấu bằng xẩy ra khi
2
2

x = y = z.
Vậy giá trị nhỏ nhất của P là

1
3
, xẩy ra tại x = y = z =
hay a = b = c = 3 .
2
3

Bài 9. Tìm số nguyên dương n lớn nhất để bất đẳng thức sau được thỏa mãn

Nguyễn Công Lợi

Trường THCS Thị Trấn Quỳ Hợp – Nghệ An


1
n

( na + b + c )

4


Tailieumontoan.com
1
1
3
+
+

4
4
n
(a + nb + c ) n (a + b + nc ) 16

1 1 1
+ + a+b+c.
a b c

Trong đó a, b, c là các số thực dương thỏa mãn
Lời giải

1

Vì bất đẳng thức
n

( na + b + c )

thực dương a, b, c thỏa mãn

4


1

+
n

(a + nb + c )

4

1

+
n

(a + b + nc )

4



3
đúng với mọi số
16

1 1 1
+ +  a + b + c nên bất đẳng thức trên sẽ thỏa mãn
a b c

3


tại a = b = c = 1 . Từ đó ta có
n

( n + 2)

4



2
3
 n ( n + 2 )  4 . Mà theo bất đẳng thức
16

AM – GM cho n số thực dương ta có
n

( n + 2)

2

= n ( n + 2 )( n + 2 ) .1...1 

Do đó 4 

n + 2 + n + 2 + 1 + ... + 1 3n + 2
.
=
n
n


3n + 2
 n  2 . Ta đi chứng minh n = 2 là giá trị cần tìm.
n

Thật vậy với n = 2 bất đẳng thức trên được viết lại thành
1

1

+

1

+

( 2a + b + c ) ( a + 2b + c ) ( a + b + 2c )
2

Ta có 2a + b + c = ( a + b ) + ( a + c )  2

Hoàn toàn tương tự ta có

Dễ thấy

2

(a + b )(a + c )

1


( a + 2b + c )

2



nên

2



3
16

1

( 2a + b + c )

2



1
.
4 ( a + b )( a + c )

1
1

1
;

.
2
4 ( b + c )( a + b ) ( a + b + 2c )
4 ( c + a )( c + b )

1
1
1
a+b+c
+
+
=
.
4 ( a + b )( a + c ) 4 ( b + c )( a + b ) 4 ( c + b )( c + a ) 2 ( a + b )( b + c )( c + a )

Do đó ta có bất đẳng thức
1

+

1

+

1

( 2a + b + c ) ( a + 2b + c ) ( a + b + 2c )

2

2

2



a+b+c
2 ( a + b )( b + c )( c + a )

Như vậy ta cần chứng minh được bất đẳng thức
a+b+c
3

 8 ( a + b + c )  3 ( a + b )( b + c )( c + a )
2 ( a + b )( b + c )( c + a ) 16

Nguyễn Công Lợi

Trường THCS Thị Trấn Quỳ Hợp – Nghệ An


Tailieumontoan.com
Để ý ta viết lại giả thiết thành ab + bc + ca  abc ( ab + bc + ca ) .
Mà ta có abc ( ab + bc + ca )

(ab + bc + ca )

Do đó ta có


( ab + bc + ca )


2

3

.

2

 ab + bc + ca  ab + bc + ca  3 .

3

Như vậy ta có 8 ( a + b + c ) 

8 ( a + b + c )( ab + bc + ca )

.
3
Dễ dàng chứng minh được 8 ( a + b + c )( ab + bc + ca )  9 ( a + b )( b + c )( c + a ) .
Do đó ta có 8 ( a + b + c ) 

9 ( a + b )( b + c )( c + a )
3

= 3 ( a + b )( b + c )( c + a ) .


Như vậy bất đẳng thức được chứng minh xong. Vậy n = 2 là giá trị cần tìm
Bài 10. Cho a, b, c là các số thực không âm. Chứng minh rằng:

a 2 + bc + b2 + ca + c 2 + ab 

3
(a + b + c )
2

Lời giải
Do vài trị của a, b, c như nhau nên khơng mất tính tổng quát ta giả sử a  b  c .
Khi đó ta có

a 2 + bc  a +

c
c2
c2
 a 2 + bc  a 2 + ac +  0  c ( a − b ) + , bất đẳng thức
2
4
4

cuối cùng luôn đúng do a  b  c  0 . Mặt khác áp dụng bất đẳng thức Cauchy –
Schwarz ta có

b2 + ca + c 2 + ab 
Như vậy ta có

(1 + 1) ( b


2

)

(

+ ca + c 2 + ab = 2 b2 + c 2 + ca + ab

(

c
a 2 + bc + b2 + ca + c 2 + ab  a + + 2 b2 + c 2 + ab + ac
2

)

)

Phép chứng minh sẽ kết thúc khi ta chỉ ra được

(

)

c
3
a + + 2 b2 + c 2 + ab + ac  ( a + b + c ) .
2
2

Bất đẳng thức trên tương đương với

(

)

2 b2 + c 2 + ab + ac 

a + 3b + 2c
.
2

Thật vậy bình phương hai vế bất đẳng thức cần chứng minh trên ta được

a 2 + 9b 2 + 4c 2 + 6ab + 12bc + 4ca
4
2
2
2
 a + b − 4c − 2ab + 12bc − 4ca  0

(

)

2 b2 + c 2 + ab + ac 

 a 2 + b 2 + 4c 2 − 2ab + 4bc − 4ab + 8bc − 8c 2  0  ( a − b − 2c ) + 8c ( b − c )  0
2


Nguyễn Công Lợi

Trường THCS Thị Trấn Quỳ Hợp – Nghệ An


Tailieumontoan.com
Bất đẳng thức cuối cùng luôn đúng do a  b  c  0 .

(

)

c
3
Từ đó ta có a + + 2 b2 + c 2 + ab + ac  ( a + b + c ) .
2
2
Vậy bất đẳng thức đã cho được chứng minh.
Dấu bằng xẩy ra khi và chỉ khi ( a; b; c ) = ( 1;1; 0 ) và các hoán vị.
Bài 11. Cho a, b, c là các số thực dương thỏa mãn a2 + b2 + c2 = 3 . Tìm giá trị nhỏ nhất
của biểu thức:
4

4


2  
2  
2 
P =  3a +

+  3b +
+  3c +


b+c 
c+a  
a + b 


4

Lời giải
Áp dụng bất đẳng thức AM – GM ta có 3a +

2
2
2a 3
= a+a+a+
 44
b+c
b+c
b+c

4


2 
2a 3
 256.
Từ đó ta được  3a +

. Hồn tồn tương tự ta có
b + c 
b+c

4

4


2 
2b3 
2 
2c 3
3b
+

256.
;
3c
+

256.

c + a 
a + c 
a + b 
a+b

Từ đó ta được
4


4

4

 a3

2  
2  
2 
b3
c3 
P =  3a +
+
3b
+
+
3c
+

512
+
+

.
b + c  
c + a  
a + b 

 b+c a+c a+b


Mặt khác áp dụng bất đẳng thức Cauchy – Schwarz ta có

(

)

2

a 2 + b2 + c 2
a3
b3
c3
a4
b4
c4
+
+
=
+
+

b + c a + c a + b ab + ab bc + ab ca + cb 2 ( ab + bc + ca )
Dễ thấy a2 + b2 + c2  ab + bc + ca nên ta có

(a

2

+ b2 + c 2


)

2

2 ( ab + bc + ca )

(a
=

2

)(

+ b2 + c 2 a 2 + b2 + c 2
2 ( ab + bc + ca )

)a

2

+ b2 + c 2 3
=
2
2

a3
b3
c3
3

3
+
+
 . Suy ra ta có P  512. = 768 , dấu bằng xẩy ra
Do đó ta được
b+c a+c a+b 2
2
khi và chỉ khi a = b = c = 1 .
Vậy giá trị nhỏ nhất của P là 768, xẩy ra khi a = b = c = 1 .
Bài 12. Cho a, b, c là các số thực dương thỏa mãn a + b + c = 1 . Chứng minh rằng:
Nguyễn Công Lợi

Trường THCS Thị Trấn Quỳ Hợp – Nghệ An


Tailieumontoan.com
1+ a 1+ b 1+ c
b c a
+
+
 2 + + 
1−a 1− b 1− c
a b c
Lời giải
Kết hợp với giả thiết a + b + c = 1 ta có

1+ a a + b + c + a
2a
. Hoàn toàn tương
=

= 1+
1−a a + b + c −a
b+c

tự ta viết lại bất đẳng thức cần chứng minh thành
3+

2a
b
c
b c a
+
+
 2 + + 
b+c c+a a+b
a b c

 2a
2a   2b 2b   2c
2c 
 −
+

+ −


3
 c b+c   a c+a   b a+ b
ab
bc

ca
3

+
+

c ( b + c ) a ( c + a ) b (a + b ) 2

Áp dụng bất đẳng thức Cauchy – Schwarz ta có

( ab + bc + ca ) .
ab
bc
ca
+
+

c ( b + c ) a ( c + a ) b ( a + b ) 2abc ( c + b + c )
2

( ab + bc + ca )  3 .
2abc ( c + b + c ) 2
2

Như vậy phép chứng minh sẽ hoàn tất khi ta chỉ ra được

Bất đẳng thức trên tương đương với ( ab + bc + ca )  3abc ( a + b + c ) , đây là một đánh
2

giá đúng.

Vậy bất đẳng thức đã cho được chứng minh.
Dấu bằng xẩy ra khi và chỉ khi a = b = c =

1
.
3

Bài 13. Cho a, b, c là các số thực dương tùy ý. Chứng minh rằng:

(

c a 2 + b2

(

)

b3 ab + c 2

2

)

+

a b2 + c 2

(

)


(
)  2 a + c + b 
+
( bc + a ) a ( ca + b )  c b a 

c3

2

2

2

b c2 + a2
3

2

Lời giải
2

Ta có

(

c a 2 + b2

(


)

b3 ab + c 2

2

)

(
(

)
)

1 2
a + b2
4
b
=
1
ab + c 2
bc

2

 a2

 2 + 1
b
 . Đặt x = a ; y = b ; z = c , khi đó ta được

=
a c
b
c
a
+
c b

xyz = 1 .

Nguyễn Công Lợi

Trường THCS Thị Trấn Quỳ Hợp – Nghệ An


Từ đó ta có

(

Tailieumontoan.com

) = ( x + 1) . Áp dụng hoàn toàn tương tự ta thu được bất đẳng
(ab + c ) x ( y + z )
2

c a 2 + b2
b3

2


2

2

(x
thức cần chứng minh là

2

) + (y

+1

2

x ( y + z)

2

+1

) + (z
2

y (z + x)

2

+1


)

2

z (x + y)

 2 ( xy + yz + zx ) .

Áp dụng bất đẳng thức Cauchy – Schwarz ta có

(x

2

+1

) + (y
2

x ( y + z)

2

+1

) + (z
2

y (z + x)


2

+1

)  (x
2

z (x + y)

2

+ y2 + z2 + 3

)

2

2 ( xy + yz + zx )

Phép chứng minh sẽ kết thúc khi ta chỉ ra được

(x

2

+ y2 + z2 + 3

)

2 ( xy + yz + zx )


2

 2 ( xy + yz + zx )  x 2 + y 2 + z 2 + 3  2 ( xy + yz + zx )

Chú ý đến điều kiện xyz = 1 ta có

(

x 2 + y 2 + z 2 + 3 − 2 ( xy + yz + zx ) = x 2 + 3 − 4 x + ( y − z ) − 2x y + z − 2 yz
2

= x2 + 3 − 4 x +

(

y− z

) (
2

y+ z

) − 2x
2

Không mất tính tổng qt ta có thể giả sử z  y  x , khi đó ta có
Lại có x2 + 3 − 4 x = x2 − 2x + 1 + 2x − 4 x + 2 = ( x − 1) + 2
2


Như vậy ta được x 2 + 3 − 4 x +

(

y− z

)

) (
2

y+ z

(

)

(

y+ z

) − 2x  0 .
2

2

x −1  0 .

) − 2x  0 .
2


Từ đó suy ra x 2 + y 2 + z 2 + 3 − 2 ( xy + yz + zx )  0 hay x 2 + y 2 + z 2 + 3  2 ( xy + yz + zx ) .
Dấu bằng xẩy ra khi và chỉ khi x = y = z = 1 .
Vậy bất đẳng thức đã cho được chứng minh. Dấu bằng xẩy ra khi và chỉ khi a = b = c .
Bài 14. Cho a, b, c là các số thực không âm thỏa mãn

( a + b + 2c )( b + c + 2a )( c + a + 2b ) = 1
Chứng minh rằng

a

b ( 4c + 15 )( b + 2c )

2

+

b

c ( 4a + 15 )( c + 2a )

2

+

c

a ( 4b + 15 )( a + 2b )

2




1
3

Lời giải
Áp dụng bất đẳng thức Cauchy – Schwarz ta có

Nguyễn Cơng Lợi

Trường THCS Thị Trấn Quỳ Hợp – Nghệ An


a

b ( 4c + 15 )( b + 2c )
2

2

Tailieumontoan.com
b
c
+
+
2
2
c ( 4a + 15 )( c + 2a ) a ( 4b + 15 )( a + 2b )
2


2

 a 
 b 
 c 
 a
b
c 
 b + 2c 
 c + 2a 
 a + 2b 
 b + 2c + c + 2a + a + 2b 
 + 
 + 
 

= 
ab ( 4c + 15 ) bc ( 4a + 15 ) ca ( 4b + 15 )
12abc + 15 ( ab + bc + ca )

2

Cũng áp dụng bất đẳng thức Cauchy – Schwarz ta có

(a + b + c )  1 .
a
b
c
+

+

b + 2c c + 2a a + 2b 3 ( ab + bc + ca )
2

Như vậy phép chứng minh sẽ hoàn tất nếu ta chỉ ra được 12abc + 15 ( ab + bc + ca )  3 .
Áp dụng bất đẳng thức AM – GM cho giả thiết ta có

 a + b + 2c + b + c + 2a + c + a + 2b 
1 = ( a + b + 2c )( b + c + 2a )( c + a + 2b )  

3



3

 4 (a + b + c ) 
4 (a + b + c )
3
 1 
 a+b+c 
  1
3
3
4


3


Cũng theo bất đẳng thức AM – GM ta lại có

2a + b + c = a + a + b + c  4 4 a 2 bc
a + 2b + c = a + b + b + c  4 4 ab 2c
a + b + 2c = a + b + c + c  4 4 abc 2
Từ đó suy ra 1 = ( a + b + 2c )( b + c + 2a )( c + a + 2b )  4 3 abc  abc 
Đặt x = a + b + c; y = ab + bc + ca; z = abc . Khi đó ta được x 

1
.
64

3
1
.
;z 
4
64

Bất đẳng thức cần chứng minh trên được viết lại thành 12z + 15y  3  5y + 4z  1.
Lại có xy = ( a + b + c )( ab + bc + ca )  9abc = 9z .
Từ giả thiết ( a + b + 2c )( b + c + 2a )( c + a + 2b ) = 1 ta được
1 = ( x + a )( x + b )( x + c ) = x 3 + ( a + b + c ) x 2 + ( ab + bc + ca ) x + abc

Hay ta được 2x3 + xy + z = 1 . Như vậy bất đẳng thức cần chứng minh trên trở thành

5y + 4z  2x3 + xy + z  5y + 3z  2x3 + xy
Ta có ( a + b + c )  3 ( ab + bc + ca )  x 2  3y  2x 3  6xy  2x 3 + xy  7xy
2


Nguyễn Công Lợi

Trường THCS Thị Trấn Quỳ Hợp – Nghệ An


Tailieumontoan.com
3
20
1
20 3
1
Do x  và xy  9z nên ta có 7xy =
xy + xy  . .y + .9z = 5y + 3z
4
3
3
3 4
3
Như vậy 5y + 3z  2x3 + xy , tức là bất đẳng thức 5y + 4z  1 đươc chứng minh.
Vậy bất đẳng thức đã cho được chứng minh. Dấu bằng xẩy ra khi và chỉ khi

a=b=c=

1
.
4

Bài 15. Cho a, b, c là các số thực dương thỏa mãn a4 + b4 + c4 = 3 . Chứng minh rằng:

1

1
1
3
+
+

6 − ab 6 − bc 6 − ca 5
Lời giải
Từ giả thiết ta có 3 = a 4 + b4 + c 4  a 4 + b4  2 ( ab )  ab 
2

Ta sẽ chứng minh bất đẳng thức

3
4
2

1
1
9
 a 2 b2 + . Thật vậy, bất đẳng thức cần
6 − ab 50
50

chứng minh tương đương với

(

)


1
1 2 2 9

a b +
 50  ( 6 − ab ) a 2 b 2 + 9  50  54 − a 3 b 3 + 6a 2 b 2 − 9ab
6 − ab 50
50

 ( ab ) − 6 ( ab ) + 9ab − 4  0  ( ab ) − 4 ( ab ) − 2 ( ab ) + 8ab + ab − 4  0
3

2

3

2

2

 ( ab ) ( ab − 4 ) − 2ab ( ab − 4 ) + ( ab − 4 )  0  ( ab − 4 )( ab − 1)  0
2

2

Bất đẳng thức cuối cùng ln đúng vì ab  4 . Dấu bằng xẩy ra khi và chỉ khi ab = 1 .
Chứng minh hoàn toàn tương tự ta được

1
1 2 2 9
1

1
9

bc + ;
 c 2a 2 + .
6 − bc 50
50 6 − ca 50
50

Cộng theo vế các bất đẳng thức trên ta được

(

)

1
1
1
1 2 2
27
.
+
+

a b + b2 c 2 + c 2 a 2 +
6 − ab 6 − bc 6 − ca 50
50
Như vậy phép chứng minh sẽ hoàn tất nếu ta chỉ ra được

(


)

1 2 2
27 3
a b + b2 c 2 + c 2a 2 +
  a 2 b 2 + b 2 c 2 + c 2a 2  3
50
50 5
Bất đẳng thức trên hiển đúng do a2 b2 + b2c2 + c2a2  a4 + b4 + c4 = 3 .
Vậy bất đẳng thức đã cho được chứng minh. Dấu bằng xẩy ra khi và chỉ khi

a = b = c = 1.
Bài 16. Cho a, b, c là các số thực dương thỏa mãn a + b + c + d = 4 . Chứng minh rằng:

Nguyễn Công Lợi

Trường THCS Thị Trấn Quỳ Hợp – Nghệ An


Tailieumontoan.com
a
b
c
d
+
+
+
2
2

2
2
1 + b c 1 + c d 1 + d a 1 + a2 b
Lời giải
Áp dụng bất đẳng thức AM – GM ta có

 a
ab ( 1 + c )
ab 2 c
ab 2 c
ab c
ab abc
=
a


a

=a−
a−
=a−


2
2
2
4
4
4
1+ b c

2b c
1 + b c
 b
bc ( 1 + d )
bc 2d
bc 2d
bc d
bc bcd

=
b


b

=
b


b

= b−

2
2
2
4
4
4
1+ c d

1 + c d
2c d

cd ( 1 + a )
cd 2a
cd 2a
cd a
cd cda
 c
=
c


c

=
c


c

=c−

2
 1 + d 2a
2
4
4
4
1+ d a

2d
a

da ( 1 + b )
 d
ab2d
ab2d
da b
da dab
=
d


d

=d−
d−
=d−


2
2
2
4
4
4
1+ a b
2a b
1 + a b
Cộng theo vế các bất đẳng thức trên và kết hợp với a + b + c + d = 4 ta được


a
b
c
d
ab + bc + cd + da abc + bcd + cda + dab
+
+
+
 4−

2
2
2
2
4
4
1+ b c 1+ c d 1+ d a 1+ a b
Cũng theo bất đẳng thức AM – GM ta có
2

a+ b+c+d
ab + bc + cd + ca = ( a + c )( b + d )  
 =4
4


Lại có abc + bcd + cda + dab = ab ( c + d ) + cd ( a + b ) . Áp dụng bất đẳng thức AM – GM
ta có
2


2

a+b
c+d
ab ( c + d ) + cd ( a + b )  
.(c + d) + 

 (a + b )
 2 
 2 
2

a+ b+c+d a+ b+c+d
= ( a + b )( c + d ) 

 =4
4
4

 


Do đó suy ra 4 −
Từ đó ta được

ab + bc + cd + da abc + bcd + cda + dab
4 4

 4− − = 2.

4
4
4 4

a
b
c
d
+
+
+
 2.
2
2
2
1 + b c 1 + c d 1 + d a 1 + a2 b

Vậy bất đẳng thức được chứng minh. Dấu bằng xẩy ra khi và chỉ khi a = b = c = d = 1 .
Bài 17. Cho a, b, c là các số thực dương bất kỳ. Chứng minh rằng:

a3
b3
c3
+
+
1
a 3 + abc + b3 b3 + abc + c 3 c 3 + abc + a 3
Lời giải
Nguyễn Công Lợi


Trường THCS Thị Trấn Quỳ Hợp – Nghệ An


Tailieumontoan.com
1
1
1
+
+
 1.
Bất đẳng thức đã cho tương đương với
3
3
bc b
ac c
ab a 3
1+ 2 + 3 1+ 2 + 3 1+ 2 + 2
a
a
b
b
c
c
Đặt x =

b
c
c
; y = ; z = . Khi đó ta được xyz = 1 và bất đẳng thức đã cho được viết lại
a

b
a

thành

1
1
1
1
1
1
+
+
1 3
+ 3
+ 3
1
2
2
x
y
z
x + x z + 1 y + y x + 1 z + z2 y + 1
3
3
3
1+ + x 1+ + y 1+ + z
y
x
z

Áp dụng bất đẳng thức Cauchy – Schwarz ta có
yz
xy
1
1
1
zx
+ 3
+ 3
= 2
+ 2
+ 2
2
2
2
x + x z + 1 y + y x + 1 z + z y + 1 x + xz + yz y + xy + xz z + zy + xy
3

=

(

( yz )

( zx )

2

yz x 2 + xz + yz


)

+

(

( xy )

2

zx y 2 + xy + xz

( xy + yz + zx )

+

)

(

2

xy z 2 + zy + xy

)

( xy + yz + zx )

=
yz ( x + xz + yz ) + zx ( y + xy + xz ) + xy ( z + zy + xy ) ( xy + yz + zx )

2

2

2

2

2
2

=1

Vậy bất đẳng thức đã cho được chứng minh. Dấu bằng xẩy ra khi và chỉ khi a = b = c
Bài 18. Cho a, b, c là các số thực dương thỏa mãn a 2 + b2 + c 2 + ( a + b + c )  4 . Chứng
2

minh rằng:
ab + 1

+

bc + 1

+

ca + 1

(a + b) ( b + c ) (c + a )
2


2

2

3

Lời giải
Ta có a 2 + b2 + c 2 + ( a + b + c )  4  a 2 + b2 + c 2 + ab + bc + ca  2 . Từ đó ta được
2

2 ( ab + 1)

(a + b)

2

=

2ab + 2

(a + b)



2ab + a 2 + b 2 + c 2 + ab + bc + ca

(a + b )
( a + b ) + ( c + a )( c + b ) = 1 + ( c + a )( c + b )
=

(a + b)
(a + b)
2

2

2

2

Hoàn toàn tương tự ta có

2 ( bc + 1)

( b + c)

2

 1+

2

( a + b )( a + c ) ; 2 ( ca + 1)  1 + ( b + c )( b + a ) .
( b + c)
(c + a)
(c + a )
2

2


2

Từ đó ta được

Nguyễn Cơng Lợi

Trường THCS Thị Trấn Quỳ Hợp – Nghệ An


Tailieumontoan.com
 ab + 1
( c + a )( c + b ) + ( a + b )(a + c ) + ( b + c )( b + a )
bc + 1
ca + 1 
2
+
+
 3+
2
2
2
2
2
2
 (a + b) ( b + c ) (c + a ) 
a
+
b
b
+

c
c
+
a
(
)
(
)
(
)


Phép chứng minh sẽ hoàn tất nếu ta chỉ ra được

( c + a )( c + b ) + ( a + b )( a + c ) + ( b + c )( b + a )  3 .
(a + b)
(b + c)
(c + a )
2

2

2

Dễ thấy bất đẳng thức trên luôn đúng theo bất đẳng thức AM – GM.
Vậy bất đẳng thức đã cho được chứng minh. Dấu bằng xẩy ra khi và chỉ khi

a=b=c=

1

3

.

Bài 19. Cho a, b, c là các số thực dương thỏa mãn a2 + b2 + c2 = 3 . Chứng minh rằng:

a4
b4
c4
+
+
1
a 2 + 2b6 b2 + 2c 6 c 2 + 2a 6
Lời giải
Áp dụng bất đẳng thức AM – GM ta có

(

)

a 2 a 2 + 2b6 − 2a 2 b6
a4
2a 2 b6
2b2 3 a 4
2
2
=

a


=
a

3
a 2 + 2b6
a 2 + b6 + b6
3 3 a 2 b12
Hoàn toàn tơng tự ta được

Khi đó ta có

b4
2a 2 3 c 4
c4
2c 2 3 b4
2
2

b

;

c

.
3
3
b2 + 2c 6
c 2 + 2a 6


 2b2 3 a 4 2a 2 3 c 4 2c 2 3 b4
a4
b4
c4
2
2
2
+
+
 a + b +c −
+
+

3
3
3
a 2 + 2b6 b2 + 2c 6 c 2 + 2a 6







Để ý rằng a2 + b2 + c2 = 3 , do đó phép chứng minh hồn tất khi ta chỉ ra được

2b2 3 a 4 2a 2 3 c 4 2c 2 3 b 4
+
+
 2  b2 3 a 4 + a 2 3 c 4 + c 2 3 b4  3

3
3
3
Thật vậy, áp dụng bất đẳng thức AM – GM ta có

(

1 2
b 1 + a2 + a2
3
1
a 2 3 c 4 = a 2 3 c 2 .c 2 .1  a 2 1 + c 2 + c 2
3
1
c 2 3 b4 = a 2 3 b2 .b2 .1  c 2 1 + b 2 + b 2
3
b2 3 a 4 = b 2 3 a 2 .a 2 .1 

(

(

Nguyễn Công Lợi

)

)

)


Trường THCS Thị Trấn Quỳ Hợp – Nghệ An


Tailieumontoan.com
Từ đó ta có

b2 3 a 4 + a 2 3 c 4 + c 2 3 b4 

(

)

(

)

(

)

1 2
1
1
b 1 + a 2 + a 2 + a 2 1 + c 2 + c 2 + c 2 1 + b2 + b2 .
3
3
3

(


)

2 a 2 b 2 + b 2 c 2 + c 2a 2
1 2
2
2
2
2
2
2
2
2 
b 1+ a + a + a 1+ c + c + c 1+ b + b
=
+ 1.
Mà ta có

3
3

(

)

Lại có a 2 b2 + b2 c 2 + c 2a 2 

(

(


)

(

1 2
a + b2 + c 2
3

)

)

2

(

(

)

= 3 . Do đó ta được

)

(

)

1 2
2

b 1 + a 2 + a 2 + a 2 1 + c 2 + c 2 + c 2 1 + b2 + b2   .3 + 1 = 3
 3
3
Từ đó suy ra b2 3 a4 + a2 3 c4 + c2 3 b4  3 .
Vậy bất đẳng thức đã cho được chứng minh. Dấu bằng xẩy ra khi và chỉ khi

a = b = c = 1.
Bài 20. Cho x, y, z là các số thực không âm thỏa mãn x2 + y2 + z2 = 1 . Chứng minh
rằng:

1

y
x
z
3 3
+
+

1 − yz 1 − zx 1 − xy
2
Lời giải

Đặt P =

y
x
z
3 3
và ta cần chứng minh 1  P 

.
+
+
1 − yz 1 − zx 1 − xy
2

• Trước hết ta chứng minh P  1 .

Mặt khác do x, y, z là các số thực không âm thỏa mãn x2 + y2 + z2 = 1 nên

0  x; y; z  1 .
Từ đó suy ra 0  xy; yz; zx  1 . Suy ra

P=

y
x
z
+
+
 x + y + z  x2 + y2 + z2 = 1
1 − yz 1 − zx 1 − xy

Dấu bằng xẩy ra khi và chỉ khi ( x; y; z ) = ( 1; 0; 0 ) và các hốn vị.
• Chứng minh P 

3 3
.
2


Cách 1. Dễ thấy yz 

Nguyễn Công Lợi

y2 + z2
y2 + z2
, do đó ta được 1 − yz  1 −
.
2
2

Trường THCS Thị Trấn Quỳ Hợp – Nghệ An


Tailieumontoan.com
x
x
2x
2x

=
=
Từ đó kết hợp với x2 + y2 + z2 = 1 ta có
.
2
2
2
2
1 − yz
y +z

1 + x2
2− y +z
1−
2

(

Hồn tồn tương tự ta được

Từ đó suy ra P 

Thật vậy ta có

)

y
2y
z
2z

;

.
2
1 − zx 1 + y 1 − xy 1 + z 2

(

)


2y
2x
2z
2x
3 3 2
+
+
. Ta đi chứng minh

x +1 .
2
2
2
2
8
1+ x 1+ y 1+ z
1+ x

(

)

(

)

2x
3 3 2

x + 1  16x2  3 3 x2 + 1 . Áp dụng bất đẳn thức AM –

2
8
1+ x

GM ta có
2

2

 2 1 1 1
1 
3 3 x + 1 = 3 3  x + + +   3 3  2 x 2 .  = 16x

3 3 3
27 



(

2

)

2

Như vậy bất đẳng thức trên được chứng minh.
Hoàn toàn tương tự ta được

P


(

)

(

(

)

)

2y
3 3 2
2z
3 3 2

y +1 ;

z + 1 . Từ đó suy ra
2
2
8
8
1+ y
1+ z

2y
2x

2z
3 3 2
3 3
+
+

x + y2 + z2 + 3 =
2
2
2
8
2
1+ x 1+ y 1+ z

Vậy bất đẳng thức được chứng minh. Dấu bằng xẩy ra khi và chỉ khi x = y = z =
Cách 2. Ta có P =

1
3

.

 1
y
x
z
1
1 
+
+

= x + y + z + xyz 
+
+

1 − yz 1 − zx 1 − xy
 1 − yz 1 − zx 1 − xy 

(

)

+ Nếu xyz = 0 khi đó ta có P = x + y + z  3 x2 + y 2 + z2 = 3 

3 3
.
2

+ Nếu xyz  0 , khi đó 0  x; y; z  1 . Khi đó áp dụng bất đẳng thức AM – GM ta có

1 − yz  1 −
Từ đó suy ra

(

)

(

)


(

)

1 2
1
1
y + z2 = 2 − y 2 − z2 = 2x2 + y 2 + z2  x2 + yz  2x yz
2
2
2

1
1
1
1
1
1
. Hoàn toàn tương tự
. Từ

;


1 − zx 2y zx 1 − xy 2z xy
1 − yz 2x yz

đó ta có

Nguyễn Cơng Lợi


Trường THCS Thị Trấn Quỳ Hợp – Nghệ An


Tailieumontoan.com
 1
1
1 

P  x + y + z + xyz 
+
+
 2x yz 2y zx 2z xy 



1
= x+y+z+
2

(

)

yz + zx + xy 

3 (x + y + z)
2




(

3 3 x2 + y2 + z2
2

)=3

3
2

Vậy bất đẳng thức dược chứng minh. Dấu bằng xẩy ra khi và chỉ khi x = y = z =

1
3

Bài 21. Cho a, b, c là các số thực dương. Chứng minh rằng:
2
2
2
a 3 − b3 b3 − c 3 c 3 − a 3 1 
+
+
 ( a − b ) + ( b − c ) + ( c − a ) 

a+b
b+c
c+a
4


Lời giải
Khơng mất tính tổng qt ta giả sử a  b  c  0 .
Bất đẳng thức cần chứng minh quy về chứng minh hai bất đẳng thức sau

 a 3 − b3 b3 − c 3 c 3 − a 3
2
2
2
1
+
+
 − ( a − b ) + ( b − c ) + ( c − a ) 


a+b
b+c
c+a
4
 3
3
3
3
3
3
 a − b + b − c + c − a  1 ( a − b ) 2 + ( b − c ) 2 + ( c − a ) 2 

 a + b
b+c
c+a
4 

• Trước hết ta chứng minh

Ta có a +

2
2
2
a 3 − b3 b3 − c 3 c 3 − a 3
1
+
+
 − ( a − b ) + ( b − c ) + ( c − a )  .

a+b
b+c
c+a
4

b a 2 + ab + b2 1
b a 2 + ab + b2

= ab − b2  0 , suy ra a + 
.
2
2
a+b
a+b
2

Lại có b +


(

)

a a 2 + ab + b2 1
a a 2 + ab + b2

= ab − a 2  0 , suy ra b + 
.
2
2
a+b
a+b
2

(

)

Như vậy ta có b +

a a 2 + ab + b2
b

a+ .
2
a+b
2


Tương tự ta có c +

b b2 + bc + c 2
c
a c 2 + ca + a 2
c

b+ ;c + 
 a + . Từ đó ta có
2
b+c
2
2
c +a
2

a 3 − b3 b3 − c 3 c 3 − a 3
+
+
a+b
b+c
c+a
2
2
a + ab + b
b 2 + bc + c 2
a 2 + ca + c 2
= (a − b)
+ (b − c)
− (a − c )

a+b
b+c
c+a

a

b

c
 (a − b)  b +  + ( b − c )  c +  − (a − c )  a + 
2
2
2



1
1
1
1
1
1
= a 2 − b2 + ab + b2 − c 2 + bc − a 2 + c 2 + ca
2
2
2
2
2
2
2

2
2
1 2
1
= − a + b 2 + c 2 − ab − bc − ca = − ( a − b ) + ( b − c ) + ( c − a ) 

2
4

(

Nguyễn Công Lợi

)

Trường THCS Thị Trấn Quỳ Hợp – Nghệ An


Tailieumontoan.com
2
2
2
a − b b − c c3 − a3 1 
• Chứng minh
+
+
 ( a − b ) + ( b − c ) + ( c − a )  .

a+b
b+c

c+a
4
3

3

3

3

Cũng từ các bất đẳng thức trên ta có

a 3 − b3 b3 − c 3 c 3 − a 3
+
+
a+b
b+c
c+a
2
2
a + ab + b
b 2 + bc + c 2
a 2 + ca + c 2
= (a − b)
+ (b − c)
− (a − c )
a+b
b+c
c+a


b

c

a
 (a − b)  a +  + ( b − c )  b +  − (a − c )  c + 
2
2
2



1
1
1
1
1
1
= a 2 − b 2 − ab + b 2 − c 2 − bc − a 2 + c 2 − ca
2
2
2
2
2
2
2
2
2
1
1

= a 2 + b 2 + c 2 − ab − bc − ca = ( a − b ) + ( b − c ) + ( c − a ) 

2
4

(

)

Bài toán được chứng minh xong
Bài 22. Cho a, b, c là các số thực dương thỏa mãn c ( c − a − c ) = a + b + 1 . Tìm giá trị lớn
nhất của:

P=

a 4 b4

( a + bc )( b + ca )( c + ab )

3

Lời giải
Từ c ( c − a − c ) = a + b + 1  ( c + 1)( a + b ) = c 2 − a  ( c + 1)( a + b ) = ( c + 1)( c − 1) .
Do c  0 nên ta suy ra được a + b = c − 1  a + b + 1 = c . Khi đó ta có
P=
=
=

a 4 b4


( a + bc )( b + ca )( c + ab )

( a + ab + b

2

3

=

a 4 b4

)(

a 4 b4

a + b ( a + b + 1)   b + a ( a + b + c )  ( a + b + 1 + ab )

)

+ b b + ab + a 2 + a ( a + b + 1 + ab )
a 4 b4

( a + b )( b + 1)( a + b )( a + 1) ( a + 1)( b + 1)

3

=

3


3

a 4 b4

(a + b ) (a + 1)( b + 1)
2

4

Áp dụng bất đẳng thức AM – GM cho các số dương ta có
4

4
 a a a   4 a3 
a3
( a + 1) =  3 + 3 + 3 + 1    4 27  = 44. 27

 

4

4

4
3
 b b b   4 b3 
4 b



( b + 1) =  3 + 3 + 3 + 1    4 27  = 4 . 27

 

4

Nguyễn Công Lợi

Trường THCS Thị Trấn Quỳ Hợp – Nghệ An


Tailieumontoan.com

Lại có ( a + b )  4ab . Từ đó ta được
2

( a + b ) ( a + 1)( b + 1)
2

Do đó P =

4

 4ab.4 4.

a 3 4 b3 49 4 4
.4 .
= .a b
27
27 36


a 4 b4

( a + b ) ( a + 1)( b + 1)
2

4



a 4 b4
36
.
=
49 4 4 49
a b
36

a b
 = =1
 a = b = 3; c = 7 .
Dấu bằng xẩy ta khi và chỉ khi  3 3
c = a + b + 1


36
Vậy giá trị lớn nhất của P là 9 , đạt được tại a = b = 3; c = 7 .
4
Bài 23. Cho a, b, c là các số thực dương thỏa mãn


1 1 1
+ + = 1 . Tìm giá trị lớn nhất
a 2 b2 c 2

của:

1

P=

5a 2 + 2ab + 2b2

1

+

5b2 + 2bc + 2c 2

1

+

5c 2 + 2ca + 2a 2

Lời giải
Ta có 5a 2 + 2ab + 2b2 = 4a 2 + 4ab + b2 + a 2 − 2ab + b2 = ( 2a + b ) + ( a − b )  ( 2a + b ) .
2

1


Từ đó ta được

5a + 2ab + 2b
2

2

1



( 2a + b )

2

=

2

2

1
.
2a + b

Áp dụng bất đẳng thức Cauchy – Schwarz ta có

1
1
11 1 1 12 1 

=
  + +  =  + .
2a + b a + a + b 9  a a b  9  a b 
Do đó suy ra

12 1
  + .
5a 2 + 2ab + 2b 2 9  a b 
1

Áp dụng hồn tồn tương tự ta có
1 2 1
1
12 1
  + ;
  + .
5b 2 + 2bc + 2c 2 9  b c  5c 2 + 2ca + 2a 2 9  c a 
1

Cộng theo vế các bất đẳng thức trên ta được
P=

1
5a + 2ab + 2b
2

Nguyễn Công Lợi

2


+

1
5b + 2bc + 2c
2

2

+

1 1 1 1
  + + 
3a b c
5c + 2ca + 2a
1

2

2

Trường THCS Thị Trấn Quỳ Hợp – Nghệ An


Tailieumontoan.com
2

 1 1 1
 1
1 1 1 1 1
Lại có  + +   3  2 + 2 + 2   + +  3 .

b c  a b c
a b c
a
Từ đây suy ra P 

3
, dấu bằng xẩy ra khi và chỉ khi a = b = c = 3 .
3
3
, đạt được tại a = b = c = 3 .
3

Vậy giá trị lớn nhất của P là

Bài 24. Cho a, b, c là các số thực dương. Tìm giá trị nhỏ nhất của biểu thức:
P=

a 3c
2 b 3a + 3bc

+

b 3a
2 c 3 b + 3ca

+

c3b
2 a 3c + 3ab


Lời giải
a
a
b
b
=
=
=
Ta có
.
3
b
c
2 b a + 3bc 2b ab + 3bc 2 ab + 3bc
2
+3
c
a
ac
a 3c

a ac

Hồn tồn tương tự ta có

Đặt x =

b 3a
2 c 3 b + 3ca


=

b
c

c
a

c3 b

.
;
=
c
a 2 a 3c + 3ab
a
b
2
+3
2
+3
a
b
b
c

a
b
c
;y =

;z =
, khi đó ta có xyz = 1 và biểu thức đã cho được viết lại
b
c
a

thành
P=

y2
x3
z2
+
+
2y + 3z 2z + 3x 2x + 3y

Áp dụng bất đẳng thức Cauchy – Schwarz và bất đẳng thức AM – GM ta có

( x + y + z ) = x + y + z  3 3 xyz = 3
y2
x3
z2
P=
+
+

2y + 3z 2z + 3x 2x + 3y 5 ( x + y + z )
5
5
5

2

Dấu bằng xẩy ra khi và chỉ khi x = y = z = 1 hay a = b = c .
Vậy giá trị nhỏ nhất của P là

3
, đạt được tại a = b = c .
5

Bài 25. Cho a, b, c là các số thực dương thỏa mãn a + b + c = 3 . Chứng minh rằng:

a2

9a + ( b + 2c )

2

+

b2

9b + ( c + 2a )

2

+

c2

9c + ( a + 2b )


2



1
6

Lời giải
Nguyễn Công Lợi

Trường THCS Thị Trấn Quỳ Hợp – Nghệ An


Tailieumontoan.com
Biến đổi tương đương bất đẳng thức cần chứng minh và chú ý đến a + b + c = 3 ta
được
a2

9a + ( b + 2c )




2

b2

+


9b + ( c + 2a )

2

+

c2

9c + ( a + 2b )

2



1
6

a
a2
b
b2
c
c2
a+b+c 1

+

+




2
2
2
9 9a + ( b + 2c )
9 9b + ( c + 2a ) 9 9c + ( a + 2b )
9
6
a ( b + 2c )

b ( c + 2a )

2

9a + ( b + 2c )

2

+

c ( a + 2b )

2

9b + ( c + 2a )

2

+


2

9c + ( a + 2b )

2



3
2

Áp dụng bất đẳng thức AM – GM cho các số dương ta có 9a + ( b + 2c )  6 ( b + 2c ) a .
2

Từ đó ta được

a ( b + 2c )

a ( b + 2c )

2

9a + ( b + 2c )

Hàn toàn tương tự ta được



2


6 ( b + 2c ) a

b ( c + 2a )

2

9b + ( c + 2a )

2

( b + 2c )

2

=

a

6

( c + 2a )

6

b

;

.
c ( a + 2b )


2

9c + ( a + 2b )

2



(a + 2b )

c

6

. Từ

đó ta có
a ( b + 2c )

b ( c + 2a )

2

9a + ( b + 2c )

2

+


2

9b + ( c + 2a )

2

+

c ( a + 2b )

2

9c + ( a + 2b )

2



( b + 2c )

a + ( c + 2a ) b + ( a + 2b ) c
6

Lại áp dụng bất đẳng thức AM – GM ta có

( b + 2c )

a + ( c + 2a ) b + ( a + 2b ) c
6
=


Như vậy ta được

a ( b + 2c )



( b + 2c )(a + 1) + ( c + 2a )( b + 1) + (a + 2b )( c + 1)
12

a + b + c + ab + bc + ca

4
2

9a + ( b + 2c )

2

+

b ( c + 2a )

2

9b + ( c + 2a )

2

+


(a + b + c )
3+

2

3

4

c ( a + 2b )

2

9c + ( a + 2b )

2



=

3+3 3
=
4
2

3
.
2


Vậy bất đẳng thức đã cho được chứng minh. Dấu bằng xẩy ra khi và chỉ khi

a = b = c = 1.
Bài 26. Cho a, b, c là các số thực không âm thỏa mãn a2 + b2 + c2 = 1 . Chứng minh
rằng:

1

a
b
c
+
+
 2
1 + bc 1 + ca 1 + ab
Lời giải

Nguyễn Công Lợi

Trường THCS Thị Trấn Quỳ Hợp – Nghệ An


×